Câu hỏi:
2 năm trước

Số nghiệm nguyên của bất phương trình \({\left( {\dfrac{1}{3}} \right)^{\sqrt {{x^2} - 3x - 10} }} > {\left( {\dfrac{1}{3}} \right)^{x - 2}}\) là:

Trả lời bởi giáo viên

Đáp án đúng: a

ĐK: ${x^2} - 3x - 10 \ge 0 \Leftrightarrow x \in \left( { - \infty ; - 2} \right] \cup \left[ {5; + \infty } \right)$

\({\left( {\dfrac{1}{3}} \right)^{\sqrt {{x^2} - 3x - 10} }} > {\left( {\dfrac{1}{3}} \right)^{x - 2}}\)

Vì \(0 < \dfrac{1}{3} < 1 \Rightarrow \sqrt {{x^2} - 3x - 10}  < x - 2\)

\( \Leftrightarrow \left\{ \begin{array}{l}x - 2 \ge 0\\{x^2} - 3x - 10 < {\left( {x - 2} \right)^2}\end{array} \right. \Leftrightarrow \left\{ \begin{array}{l}x \ge 2\\{x^2} - 3x - 10 < {x^2} - 4x + 4\end{array} \right. \Leftrightarrow \left\{ \begin{array}{l}x \ge 2\\x < 14\end{array} \right.\)

Kết hợp nghiệm ta có: \(x \in \left[ {5;14} \right)\)

Vậy bất phương trình có 9  nghiệm nguyên.

Hướng dẫn giải:

\({a^x} \le {a^y} \Leftrightarrow \left[ \begin{array}{l}\left\{ \begin{array}{l}0 < a < 1\\x \ge y\end{array} \right.\\\left\{ \begin{array}{l}a > 1\\x \le y\end{array} \right.\end{array} \right.\)

Câu hỏi khác